You are on page 1of 61

The Hard Facts About Your GMAT Score

See how the chart below shows GMAT scores in 3 Tiers? I think people generally have a vague
notion that 3 tiers exists, but with no hard evidence like this chart... who's to say otherwise

Here's a follow-up to the last chart. Again 3 tiers of applicants regarding GMAT scores.
Here's an interesting graph from Kellogg's 2004 entering class. This is a good GMAT question
possibility. If 5282 applied to the 2-year MBA class resulting in 469, what percentage of
students were admitted with the various GMAT brackets of scores as shown in the graph?
When you do the math (Taking into account that Kellogg admits 12% more than needed for
instances where the applicant chooses another school), you get the following: 640 or less
GMAT = 1056 apps with 42 accepted at 4% acceptance. 650-690 GMAT = 1584 apps with 158
accepted at 10% acceptance. 700-740 GMAT = 2059 apps with 252 accepted at 12.2%
acceptance. 750-780 GMAT = 581 apps with 74 accepted at 12.7% acceptance. That means
the 2004 class had 25% of all students who scored 700 or more accepted. However, the actual
student population is actually 62% people who scored 700 or more. This is actually a great
challenge and encouragement as I thought it was much worse. So 38% of the class scored 690
or less? That's cool. 700-740 is only 2% more likely to receive acceptance than those who had
650-690. Not much of a difference. I think this is why we ought not to kill ourselves for not
getting 700 and for not killing our self if we do get 700 or more and don't get into a top Business
School. That 2% can't hurt though.

Comments from a GMAT Instructor


Last year, I received some feedback from a 12 year veteran Princeton Review GMAT instructor.
Here are some comments he wrote in an e-mail to me that may be helpful for you.
What we do for high scoring students to make them get their scores up (in a classroom
setting) is to run classes that focus on ONE AREA. For instance, Statistics and GMAT
Miscellany, SC, CR, Data Sufficiency. People at or above the 600 level have to revamp their
study plan. Random info dump won't be effective anymore. If you can get a 600, then you
basically know most of the information there is on the test. What you are probably lacking is a
systematic way of using that information and an understanding of good GMAT strategy.

I told the instructor that I wanted to focus on probability a bit more. His reply on that
was as follows:
You are basing some of your plans and concerns on forums. In the last 5 "Test Recons" there
have been an average of 1.3 probability questions per test. 1.3 ... think of all the energy and
time dedicated to probability in the forums and think of what kind of wasted effort is poured into
that subject. Forums are 95% noise, 4% good intention, and 1% information. If you think of it
that way you will actually look at them with an entirely new eye and perhaps glean the little
there is there to be had. Test prep companies sometimes fall for the same sort of paranoia.
They respond to the concerns of their students and when those students' fears are being fed
and influenced by the forums a nasty little spiral starts to form. Ask how many people teaching
the GMAT took the GMAT in the last 6 months I think you will be shocked and dismayed. While
I do not think that someone has to take the test often to be a good teacher, a good teacher will
take the test often to be able to screen the noise.
The graphic below is part of how I assess student needs. Each red zone is a plateau. It is a
score range that is REALLY hard to break through. You are in the plateau that the course is
designed to get people to. Don't believe the nonsense out there. There is no super effective
one-size-fits-all solution to maximum GMAT improvement. Achieving and moving through
each plateau has its own unique set of needs and concerns.

I found about 19 people who scored 700+ and found quite a few similarities in their study habits
and what books they chose to prepare with. Here is a summary of that analysis.

Books used (98% used the following)


1. Kaplan GMAT book
2. Princeton Review book
3. Official Guide for GMAT
4. Power Prep Tests
5. Manhattan GMAT Study Guide Books (New!)
Book/Resources used (About 4-5 people used the following)
1. Kaplan 800 - Hard GMAT questions - more than a few argued that this had the hardest
questions they found.

2. 800-score tests – A good resource for tests.


3. GMAT Plus – A good resource for tests.
4. DeltaCourse - primarily a combination, probability, permutation advanced study guide
with plenty of sample questions to practice with.

5. Barron's - All said study guide part sucked, but questions were useful.
6. Verbal Workout for GMAT -Princeton

Books/Resources all said sucked.


1. Arco Master the GMAT CAT – Many of the answer are wrong. But the study guides are
good.

2. Petersons
3. Crack-GMAT – Don’t waste your time or money on this one according to many.
4. Kaplan’s in-person GMAT prep course – a few who did it said that it didn’t help.

Study tips that every single 700+ scorer mentioned.


1. Record all mistakes on wrong questions and revisit them no sooner than 5 days. If still
getting wrong, then zero on the subset's and go for more help.
2. Always read the explanation in the book to see why you get it wrong and take enough
time to really understand the concept.

3. Pick a date and register. Don't study then register. There was a tendency to not take prep
time as seriously.

4. Quite a few went through OG questions more than once. Not all questions on the repeat
though; Mainly verbal sections and Quant that were areas of weakness.

5. Wrote practice essays at least 20 times under timed conditions.


6. Visit the test center before the day of test. Many did so a week or two before.
7. Time spent by most averaged between 3 and 4 months.
8. All averaged 2-3 hrs per day and no less than 10-12 hrs on the weekends.
9. Took advantage of every break during the GMAT. Take a bathroom break, some washed
their face to wake up, got fresh air.

10. Study in blocks. One person mentioned studying in 80 minute blocks without getting up to
build stamina. Another person mentioned studying in 75 minute blocks with exact
numbers of questions that are on the real GMAT to build stamina and skill. Many agree
that the ability to handle the time constraints is nearly as important is solving the problem.
Scores in the actual GMAT are worse if you leave an answer blank than if you get the
answer wrong.

Interesting note:
The Official Guide for GMAT contains questions that are in the 550 to 650 difficulty range.

Problem Solving and Data Sufficiency Study Strategy


Kaplan and Princeton are good for most topics except for Combination, Permutations,
Probability and Counting. They don’t cover these Quantitative math skills very well. The best
resource I’ve found is from Deltacourse Their material is highly focused on these topics. It was
extremely helpful. For $27, I wouldn’t even hesitate about ordering it online. You can’t buy it in
bookstores. For a scaled down version of this, go to Appendix A of this Guide.
For every Data Sufficiency and Problem Solving question you get wrong, write out in long
hand the solution. Don’t just write out the formula. This will help you process the theory being
tested. Explain why you did certain steps. I call this the teaching method explanation.

Here’s a number Property table from Kaplan Workshop that neither the Kaplan or Princeton
books provided and was extremely helpful as I realized that there were number properties I didn’t
remember or recall having learned years ago.

Number Properties
Integer Positive Negative Even Odd Prime
0 X X
1 X X X
2 X X X X
7 X X X X
-3 X X X
.5 X
Pi X
√6 X
-19/2 X
51/3 X X X X
Even negative x Odd negative X X X
Odd negative + Odd negative X X X
Even Positive – Odd Negative X X X
Odd negative / Even positive X

Points of emphasis:
O is an integer, it is neither pos of neg.
Negative numbers are not prime
0 and 1 are not prime

Reading Comprehension Study Strategy


1) 1) Aggressively read each paragraph for its main idea. If you can’t write down in a few
words what the point of each paragraph is, you weren’t reading actively enough. You
should jot down the following.
a. a. Main idea or primary purpose

b. b. Organization/Structure

c. c. Tone or attitude of author (if applicable)

Note: Be careful to not write facts down. It’ll bog you down and usually results in a loss of
the big picture and moves you to focus to much into the details.

2) 2) Note any trigger words, same train of thought words, yin-yang parallelism. See the
Princeton Review book for more explanation.
3) 3) Weed out possible disputable answers. Vague, wimpy answers are often correct
over stronger statements. ETS doesn’t want to get many complaints that a particular
answer that was strongly stated, that exceptions could arise. ETS would rather play it
safe.
4) 4) Minority passages are often positive in tone and answers tend to be positive in tone
as well. Again, ETS, wouldn’t want to look prejudiced.
5) 5) Always eliminate bad choices first before answering. You’ll almost always be able to
narrow down to 2-3 and that significantly improves your odds of getting the question right.
6) 6) Read the entire passage before answering the questions. Other books say skim, but
it’s not always successful with more difficult passages where minute details change
meanings of the passage and could get you going down the wrong path on inference or
main point questions. Give yourself 3 minutes or less.
7) 7) For Inference questions, (Note: these are usually the hardest of all RC questions) go
find the general area being referenced. Read a bit above or below it and then make your
choices. Don’t go by memory. This is going to cause more problems that be helpful with
saving time in the long run. Your answer should never contradict the main point of the
passage.
8) 8) Most people get main point and inference questions wrong so focus more carefully
on these.
9) 9) "According to the passage/author" question type of questions. Whenever you see
this question, tell yourself, "Stop and stop thinking. I need to FIND, not think."
10) 10) For main point or central idea type of questions, re-read the first and last sentences
of each paragraph before making elimination choices and answering. Getting the overall
structure is really helpful before answering.
11) 11) In Summary: Consider weeding out answers that
a. a. Are disrespectful to others/professionals. ETS doesn’t like to be
disrespectful.
b. b. Too strong an answer. Use of words like “only”, “definitely”, “positively”

c. c. Condone/approve prejudicial attitudes. ETS doesn’t like to be disrespectful.

12) 12) For a much more detailed strategy for Reading Comprehension, go to Appendix D

Sentence Correction Study Strategy


Overall strategy
1) 1) Memorize the 8 Princeton Review methods recognize bad sentences. I’ve
summarized the key points below. The Kaplan Workshop, Princeton Review/Kaplan and
Arco’s Sentence Correction study sections are the key resources that I’ve learned the
most from. And believe me… I had a lot to learn.
2) 2) Initially, do 20-30 questions and explain what error you saw in the sentence or didn’t
see. For wrong answer choices, list/explain specifically what Princeton Review rules this
answer choice violated. Sometimes, it’s a combination. Also explain why the sentence
was tricky if it’s a difficult one. For example try to identify if there were long prepositional
phrases between a verb and subject agreement or a long adjective modifying phrase
instead of an easy single word modifier.
3) 3) For all future questions, make sure you write, verbally, or mentally talk through the
rule that was violated. Make notes on a scratch piece of paper. If you pick an answer and
can’t explain EXACTLY what method the answer corrects, then you are just guessing or
going off of what sounds good--- Therefore, reinforcing a bad habit.

Spotting bad sentences is the key to doing well on sentence


structure test questions.
1) 1) Pronoun error
a. a. Plural and single – once you start with one, you need to stay in the same
quantity.

Singular Pronouns (Memorize these. Period)


Hint: Do you see the categories I setup? It’s SANE to memorize this
Some---
Any—
No—
Every--
Everyone Someone Either One
Everybody Somebody Neither Each
Everything Something
Anyone No one Whoever His
Anybody Nothing Whomever
Anything Nobody

Be aware that group, jury, team, country, family are singular. Society today uses
them sometimes as plural. This is because these act as a single unit when they
do something.

Plural Pronouns (Memorize these. Period)


Both Their Many Several
Few Others

To memorize this, try Frightening TOMBS


Singular and Plural Pronouns – depends on whether the noun is singular or
plural (Memorize these. Period)
Some More Most All

The plural and singular clause error


When two nouns are in the sentence doing an action together but they are
linked with
Along with
Together with
With
As well as
In addition to
Accompanied by
… this does not make the following action they do plural. Only “and” can take
the two singulars and make their action plural.
For example
Janie, with her poodle limping behind her, walks to the dog park.
Explanation: Janie is singular. The poodle is singular. They both do the action
together, but the use of “with” means that we need to keep the verb singular.
“Walks” is singular and “Walk” is plural
Remember, a verb that ends with an –s is singular.
b. b. Pronoun reference error- referring pronoun is not correctly placed.

c. c. Relative pronouns are often used incorrectly today.

1) 1) Referring to things or animals – that, which

2) 2) Referring to people—who, whom

3) 3) They – be careful that you don’t use this unless you’re positive
there is a referring noun. Today we often use “they” to replace the use
of a proper noun which it is not. It’s a Pronoun.

2) 2) Misplaced Modifier (modifiers must stay close to home)


a. a. Definition: Sentences that begin with a verb, adjective+verb, adjective
phrases need to be followed by the noun or pronoun they are modifying. Usually
end with –ing.
b. b. Example: Coming out of the department store, John’s wallet was stolen.
“Coming” is the modifier. Was john’s wallet coming out of the store? Incorrect
sentence.
Possible solution to look for:
a) a) Correct the reference

b) b) Put a noun or pronoun into the 1st part of the sentence turning
the 1st part into an adverbial clause.
c) c) Correct answer might be, “As he was coming out of the
department store, John’s wallet was stolen.” Alternate could be, “As
John was coming out of the department store, his wallet was stolen.”
Absolute Phrases: Introduction
1. Introduction
Definition and rules. An absolute phrase is a modifier (quite often a participle), or a
modifier and a few other words, that attaches to a sentence or a noun, with no
conjunction. An absolute phrase cannot contain a finite verb.
Absolute phrases usually consist of a noun and a modifier that modifies this noun,
NOT another noun in the sentence.
Absolute phrases are optional in sentences, i.e., they can be removed without
damaging the grammatical integrity of the sentence. Since absolute phrases are optional
in the sentence, they are often set off from the sentence with commas or, less often, with
dashes. We normally explain absolute phrases by saying that they modify entire
sentences, rather than one word. This is an important concept, since many similar
phrases that we work with modify other words. For example, adjectives modify nouns,
and adverbs can modify verbs, adjectives, and other adverbs. That said, however, in
some cases, it seems to make more sense to say that absolute phrases modify nouns.
We will look at some of these examples a bit later.

First, let's look at some examples of absolute phrases:


Examples of Absolute Phrases:
The absolute phrase look like this:
• • Her determination stronger than ever, Nexisa resolved not to give up until
she had achieved her dreams.
• • The sun shining bright and the pale blue sky forming a backdrop of the
Sacre Coeur, Carl stepped into his future as a traveler and observer.
• • We finished the hearty meal quickly, our appetites satisfied, our minds at
peace.
• • All things being equal, the active voice tends to be correct more often than
the passive on standardized tests.
Please notice that in every case the absolute phrase provides some sort of information
that works to put the whole sentence or idea in context. Please also notice that the
absolute phrases themselves do NOT contain verbs, nor are they connected to the main
sentence with a conjunction. Finally, please notice that the primary components of most
(but not all) of these absolute phrases are a noun + a modifier, although it is possible to
use only a modifier.
Here is the next pattern we should know:

2. noun + participle
This is one of the most common ways to form an absolute phrase. It might be helpful for
some people to imagine this pattern with a verb between the noun and the participle. For
example, if you say The question was still unanswered, you have a complete sentence; if,
on the other hand, you say The question unanswered and you then attach that phrase to
a main sentence, then you have an absolute phrase.
Here are some examples. The absolute phrases look like this.
• • The question still unanswered, the teacher decided to address the confusion
of her students more closely.
• • The train running late, we decided to get off at the next stop and take a taxi
home.
• • There are many industries in California vital to its economy, with technology
being one of the most important.
Compare these sentences with the verbs and conjunctions in them:
• • The question was still unanswered, and the teacher decided to address the
confusion of her students more closely.
• • The train was running late, so we decided to get off at the next stop and take
a taxi home.
• • There are many industries in California vital to its economy, and technology
is one of the most important.
Similarly, having + past participle is often so semantically similar to the sentence without
it that many sentences are written without having + past participle.
An example would be very good here:
• • Having been chosen to head the committee, Angus Ng thought about how
he could help raise money for his chess club at Harvard.
This sentence could look like this:
• • Chosen to head the committee, Angus Ng thought about how he could help
raise money for his chess club at Harvard.
This concept is important for the Sentence Correction section of the GMAT, so if you're
preparing for that test, pay attention to this!

3. noun + adjective
Another pattern is to use an adjective after the noun it modifies.
Look at these examples:
• • Their meal still not ready after 45 minutes, the hungry and angry customers
left the restaurant.
• • His hat in hand and pride in check, Horace asked his former boss for his job
back.
• • The previews still showing, Kelly and Chris decided to leave the theater and
enjoy the sunny day.

3) 3) Parallel Construction
a. a. Is there a list?

b. b. Is the sentence in two parts?


Both types must have parallel types of verbiage.
to ____, to ____ or ate _____, slept ____, drank ____.
Bad construction might look like
to ____, _____ or ate _____, sleep _____, drank ____.
c. c. Comparisons must be logical
Find the two things being compared and see if the sentence is structured in
balance. Don’t be afraid to consider changing verbs or adjectives to get the
balance.
d. d. Parallelism is not just about clauses, but verb usage
Example: -ing and –ing, to…… to…… , either ….. or, neither ……. nor.
e. e. The words "like," "unlike," "similar to," "as…so", “when” and "in contrast to"
are the most common indicators of comparisons. In comparisons, compatibility
is determined by subject matter. For example:

As domesticated animals, indoor cats typically lose their ability to hunt for their
own food, so too do domesticated dogs come to rely exclusively on their owners
for sustenance.
Here, domesticated cats are compared to domesticated dogs, and the
comparison works because they are both domesticated animals — they are like
terms. Whenever you see a comparison being set up in a sentence, check to see
that the terms of the comparison are compatible.
4) 4) Verb Tense

a. a. Is the whole sentence in the same tense

b. b. Some major categories of tense: Don’t need to memorize types. Just be


familiar
Present tense example: He walks three miles a day.
Simple Past example: When he was younger, he walked three miles a
day.
Present Perfect example: He has walked.
Past Perfect example: He had walked.
Future example: He will work.

Present Perfect – Describes action that began in the past but continues until the
present
Key identifier – “has” “have”
Sidenote: Sometimes used when deadline exists.
Past Perfect – Describes action that started and stopped in the past
Key identifier – “had”
Present Progressive – Used as emphasis by the speaker that the action is happening
this very minute
Key identifier – verb ‘to be’ + a verb with an –ing ending.
Perfect Progressive – Occupies more than one moment in the past. In other words,
ongoing for a period of time.
Key identifier – “had been”
c. c. Two events that have taken place , are taking place or will take place at the
same time must have the same tense in the sentence.
d. d. Passive verbs begin with the form of “to be” (Example: to be, were, was) and
end with a different verb in the past tense.

5) 5) Subject-Verb agreement errors.

a. a. Singular + Plural agreement. Do the two agree in plurality? Can be made


very complex when prepositional phrases separate verb from noun/subject by 5
or more words. Easy to overlook cross referenced subject-verb relationship.
b. b. To tell if a verb paradigm is plural or singular.

1) 1) Mentally put “They” in front of the verb ß plural

2) 2) Mentally put “He” in front of the verb ß singular

c. c. Verb plurality: (This comes up A LOT on the GMAT)


Adding an “s” to the end of an adjective makes it singular.
For example, dislike ß plural dislikes ß singular
Subject/Verb Inversion
So you already know that the GMAT test is an adaptive test, meaning that your score
goes up or down depending on the difficulty of the questions that you answer correctly or
incorrectly. On the sentence correction section of the GMAT, the questions that test you
on subject/verb inversion tend to be the harder questions, and are therefore worth more
points.
So, to raise your GMAT score, you should be very familiar with most or all of the items on
this list!

There are at least eighteen types of inversion:


Type Examples Notes

1. neg intro Never do I sleep. Question form is obligatory.


Only at night can I study. Used with all verbs.
In no way could I help you with This one is very common on the
your Japanese grammar TOEFL and somewhat common on
question. the GMAT and GRE.
I believe that only rarely will I We need to learn the various types
need your help. of words and phrases that require
Not until I got home did I realize this type of inversion.
that my shoes were untied. Notice that sometimes the inversion
occurs right after the neg intro form
and sometimes it occurs in the next
subject and verb.

2. intro adverbial Into the room ran the lady. Inversion is optional.
First comes love, then comes Used with be-verbs, linking verbs,
marriage. and verbs of direction.
After A comes B, then comes C, This one is less common on the
next comes D. TOEFL, but more common on the
Down came the rain and GMAT and GRE.
washed the spider out. Notice that sometimes we have an
adverb, like first and down and
sometimes we have an adverb
phrase like into the room or after A.
These adverbs and adverb phrases
usually show location or direction.
This type of inversion usually only
occurs with be-verbs, linking verbs
and verbs that show direction or
movement, like come, go, run, etc.

3. intro –ed Found in San Francisco is Inversion is obligatory.


Lombard Street, the so-called Used with be-verbs.
crookedest street in the world. This one is very common on the
Lost among the old tables and TOEFL, GMAT, and GRE.
chairs was the priceless This type of inversion usually occurs
Type Examples Notes

Victorian desk. with be-verbs, but sometimes with


Located between San Francisco linking verbs.
and Marin County is the Golden Notice that the phrase is the
Gate Bridge. complement of the be-verb.

4. comparatives Cheetahs run faster than do Inversion is optional.


antelopes. Used with all verbs.
You speak Chinese better than This form of inversion is common on
do I. the TOEFL, GMAT, and GRE.
Jessica is more interested in We normally only have inversion
Computer Science than is here if we are comparing subjects of
Benjamin. the verb, not objects. For example,
in the following two sentences, we
are comparing objects, carrots and
potatoes, not the subject I.:
I like carrots more than I do
potatoes.
I like carrots more than do I like
potatoes.
Now, in this sentence, we are
comparing subjects, I and my friend
Carl:
I like carrots more than does my
friend Carl.

5. intro Bigger than an apatosaur is the Inversion is obligatory.


comparative blue whale. Used with be-verbs.
More important than your This form is more common on the
personal statement is your GPA. GMAT and GRE than it is on the
No less impressive than the TOEFL.
invention of the laser was the Notice that we can only use this form
development of the wheel. of inversion when the verb is a be-
verb since in every case, the
comparative is the complement of
the be-verb.
Remember that less than is also a
Type Examples Notes

comparative.

6. as Megumi is from Japan, as is Inversion is obligatory.


Sato. Used with all verbs.
So-eun wants to leave early We can only use inversion if we are
today, as does Oi. using as for comparisons.
If thrown into the water, camels as is one of the trickiest words in
can swim, as can cats. English; it can have many different
meanings.

7. so… that… So happy was I that I bought Question form is obligatory.


flowers for everybody in class. Used with all verbs.
So quickly did she leave that we This is not so common on the
did not even realize was gone. TOEFL, but is fairly common on the
So rarely does a comet appear GMAT and GRE.
visible to the naked eye that The so… that… clause must before
when one does, it is considered the verb in for this type of inversion.
a major event.

8. had, should, Had I remembered Tomomi’s Inversion is obligatory.


were for if-clauses birthday, she wouldn’t be mad at Used with all verbs.
me now. This is somewhat common on the
Should you need a hand, I will TOEFL and more common on the
be more than happy to help GMAT and GRE.
you. This type of inversion is kind of
Were I you, I think I would study special. Notice that we can only use
more for your exam tomorrow. this type of inversion when we are
using an if-clause. In other words, if
is omitted: even though the word if
does not appear in the clause, we
still have the meaning of an if-
clause.
For more information, see had,
should, were.

9. there is, there There is a good restaurant Inversion is obligatory.


are, there exists, nearby. Usually used only with these verbs.
Type Examples Notes

there comes, etc. There comes a time in every This form of inversion is common on
person’s life when she realizes the TOEFL, GMAT, and GRE, as
that she is responsible for her well as in spoken and written
own happiness, not other English.
people. Most people remember there is and
Scientists hypothesize that there are. BUT we must also
there exists a certain type of remember that there are other verbs
particle that can travel faster that we can use instead of is and
than the speed of light. are. The most common ones are
exist, come, and go.

10. here is, here Here is some good food for you Inversion is obligatory.
are, here comes, to try. Usually used only with these verbs.
here come Here are the books that I don’t You will probably not see this on the
need anymore. grammar section of the TOEFL or on
Here comes the bus! the GMAT or GRE. It could,
however, appear on the Listening
Comprehension Section of the
TOEFL. We use this form mostly in
spoken English.

11. intro -ing Burning out of control was the Inversion is obligatory.
forest located in the foothills of Used only with be-verbs.
the Sierra Nevada mountains. This form is not common on the
Coming in last in the race was TOEFL, but might show up on the
Joe “Elephant Legs” Blow. GMAT or GRE.
Not helping the situation was Notice the intro –ing phrase is the
little Susie, who was throwing complement of the be-verb.
newspaper on the spreading
fire.

12. emphasis Boy am I hungry. Inversion is optional.


Is it ever hot in here! Used with all verbs.
Do you know how to cook! You will probably not see this on the
grammar section of the TOEFL or on
the GMAT or GRE. It could,
Type Examples Notes

however, appear on the Listening


Comprehension Section of the
TOEFL. We use this form mostly in
spoken English.

13. the bigger, the The closer an object is to Question form is optional.
better another object, the greater is Used with all verbs.
the gravity between the two
objects.

14. questions Is this the last example? Inversion is obligatory.


Do you enjoy reading these Used with all verbs.
lists? You will probably not see this on the
Are we finished yet? grammar section of the TOEFL
(TOEFL doesn’t test questions
anymore) or on the GMAT or GRE. It
would, however, appear on the
Listening Comprehension Section of
the TOEFL.

15. "story speech" “I think it’s time to go,” said Inversion is optional.
Susan. Used with verbs that report speech.
“It’s time for you, but not for You will probably not see this on the
me,” replied Gary. grammar section of the TOEFL or on
“Maybe we should collect our the GMAT or GRE.
thoughts for a moment,”
commented Lany.

16. nor No one has volunteered for the Inversion is obligatory.


job, nor do we expect anyone to Used with all verbs.
volunteer in the future. You might see this on the adaptive
Hok-ming cannot speak TOEFL if you are scoring high and it
Portuguese, nor can José could appear on the GMAT or GRE.
speak Cantonese. Remember that nor is considered a
The zoo regulations will not conjunction, but we use it between
permit you to touch the animals, two sentences (not between any two
Type Examples Notes

nor would most people advise elements like the other


you to do so. conjunctions).

17. "so do I"/ “So do I.” Inversion is obligatory.


"neither do I." “So can Terry.” Used with all verbs.
“Neither do most people I You will probably not see this on the
know.” grammar section of the TOEFL or on
the GMAT or GRE.

18. intro adjective Beautiful beyond belief was my Inversion is obligatory in most cases.
baby daughter. Used with be-verbs.
Happy about their acceptance This one is fairly rare and probably
into their dream schools were would not appear on the TOEFL, but
Lany and Tomo. you might see it on the GMAT or
Quick and painless will be your GRE.
medical procedure. Inversion is sometimes not used in
poetic language.

6) 6) Parallelism (Apples + Oranges)

a. a. When the sentence compares two items. Ask yourself, can they be really
compared?
b. b. When the sentence compares two actions as well.

Usually, the problem is with hidden comparison where two things or actions are
compared, but another two items or actions are intertwined and you lose the
comparison relationship.
Example: Synthetic oils burn less efficiently than natural oils.
The sentence is wrong because we are trying to compare well each oil burns and
not the oils themselves. But do you see how the actual thing being compared is
easily missed?
In a series of two or more elements, what you do on #2 determines what you do
on 3+. In other words, everything after #2 must match #2:
• • I like to swim, to run, and to dance.
• • I like to swim, run, and dance.
are okay.
• • I like to swim, run, and to dance.
• • I like to swim, to run, and dance.
are NOT okay.

7) 7) Quantity Words

a. a. The words measuring quantity may be used incorrectly. For example, when
comparing two items, it would be inappropriate to use “among” to compare them.
Here’s a chart:
2 items if more than 2
Between among
More most
Better best
Less least
b. b. Items that can’t be counted should not use quantity words. For example,
you can’t say “fewer soup”. Here’s a chart:
Ok words for non-countable Countable words
Less fewer
Amount, quantity number
Much many
c. c. When two distinct words or phrases are joined by the correlatives either, or,
neither, nor, not only, but also, the number( singular or plural) of the word or
phrase nearest to the verb determines the number of the verb.
1) 1) Example: Either his parents or he is bringing it (notice “is” is
singular) This can be a confusing sentence because parents is plural,
but we pay attention to he which is the noun “he” tells us that we need
to keep “is” singular.
2) 2) Example: Either he or his parents are bringing it. Notice “ parents”
is plural and since this is the noun that is closest to the verb, we use
“are” which is plural.
8) 8) Idioms

• • Definition: Incorrect usage of idiomatic expressions. There are no rules. Really


need good English familiarity.

• • Overall rule: If it’s not one of the previous 7, then it’s very likely an idiom
expression test question.

• • There are three types of idioms that you'll see on the GMAT:
1. 1. word pairs that go together

2. 2. prepositions and the verbs that use them

3. 3. standard expressions.

Idiomatic preposition usage


Many idiomatic expressions tested on the exam involve prepositions. There's no
overarching grammatical rule that tells you which prepositions go with which verbs.
Again, the rules are determined by usage, so you'll have to "listen" to the expression and
determine if the verb is followed by the correct preposition.
Be on the lookout for commonly tested prepositions like "of," "at," "by," "in," "from,"
"to," and "for". If you have difficulty determining whether a usage is correct, try testing out
the idiom in a simpler version of the sentence.
Example:
Although he was considered as a leading proponent for the controversial new
initiative, the professor nevertheless sought refuge from the media uproar.
The sentence above becomes:
The professor was considered as a proponent.
Does anything sound unusual? Could this sentence be worded differently? In fact, to be,
not as, is the correct idiom:
The professor was considered to be a proponent.
Would rather ß present tense if referring to myself and past tense if someone other than
the subject is doing the action
Example: Would rather I speak ß present tense
Example: Would rather you spoke ß past tense

Major Idioms you should be familiar with.


a debate over more X than Y ...(SC-175,pg 737 of OG)
a lot more...than ever...(SC-98,pg 718 of OG)
a responsibility to must have (done)
a result of neither...nor
a sequence of no less....than
accompanied by noted that ..(Sc-258,pg 759 of OG)
affect to.. (SC-47,pg 707 of OG) not in a flash...but in a ... (SC-22,pg 701 of OG)
agree with not only...but also
alternative to not so much...as
as a result of...(SC-263,pg 761 of OG) not X ...but rather Y ..(SC-172,pg 736 of OG)
as an instance of one attributes X (an effect) to Y (a cause) (SC-21,pg 701
as good as/or better than of OG)
as great as One X for every ZZ( some numeric number) Y's ...(SC-
as good as...or better than 89,pg 717 of OG)
as much as..(SC-105,pg 720 of OG) persuaded X to do Y (sc-173,pg 736 of OG)
assume ...to be of... prohibits X from doing Y
attend to (someone) potential to
attribute X to Y/X is attributed to Y range from X to Y ...(SC-58,pg 710 of OG)
based on range of ...(SC-77,pg 714 of OG)
believe X to be Y reason….. that incorrectly seen as reason….. because
benefit from regards X as Y ...
better served by X than Y ..(SC-116,pg 722 regard as
of OG) regardless
between X and Y (Sc-115, pg 722 of OG) replacing with...
both X and Y research to (SC-112, pg 721 of OG)
call...to consider... responsible for
centers on resulting in
concerned with restitution...for ...(SC-130,pg 726 of OG)
conform to retroactive to
contrary to... same to X as to Y (sc-54,pg 709 of OG)
created with seem...to...(seem is plural)
credited with ..(SC-111,pg 721 of OG) seem to indicate ...(SC-95,pg 718 of OG)
decline in so X as to be Y
defined as so (adjective) that
depends on whether so X that Y ...(SC-177,pg 737 of OG)
depicted as subscribe to
different from/differ from such...as
distinguishes between X and Y (SC-15,pg targeted at ( SC-195,pg 742 of OG)
699 of OG) that X ...that Y ...(SC-250,pg 757 of OG)
distinguish from the same to X as to Y
doubt that to …….. used to (example to get used to or to become
either...or used to)
enable to to contrast X with Y
estimated ...to be ...(SC-14,pg 699 of OG) to monitor ...(SC-239,pg 754 of OG)
entrusted with to mistake X for Y (sc-150,pg 730 of OG)
expected X to be Y to orbit...(SC-49,pg 708 of OG)
expected that X would be Y ... to result in
extent to ...(SC-57,pg 710 of OG) to sacrifice X for Y
fascinated by to survive (SC-181,pg 738 of OG)
forbid X to do Y identical with used to (do)
forcing ...to... used in the construction...
for jobs..(SC-109,pg 721 of OG) used to construct...
for over...XXX years... viewed marriage as ( sc-147,pg 730 of OG)
from X to Y widely anticipated that....
had better(do) X as Y ..(SC-182,pg 738 of OG)
in contrast to X is attributed to Y (SC-21,pg 701 of OG)
independent from X forbids Y to do Z ...(SC-100,pg 719 of OG)
indifferent towards X [is] expected to Y (SC-131, pg 726 of OG)
interaction of ...(SC-237,pg 753 of OG) X ordered that Y be Z'ed...(SC-218,pg 748)
modeled after X ordered Y to do Z (sc-121,pg 724 of OG)
(no) more...than/(no) less...than X ordered Y to be Z'ed..(SC-218,pg 748 of OG)
X prohibits Y from doing Z ...(SC-100,pg 719 of OG)

Examples of Idioms in Sentences


A
access to The company has access to large capital reserves.
act as The poison pill in the contract acts as a preventative measure against hostile
takeovers.
allows for The design of the robot arm allows for great flexibility.
as....as Chocolate tastes as good as ice cream.
associate with He associates beer with potato chips.
attribute to The poor first quarter results are
attributed to the restructuring.
a responsibility to The CEO has a fiduciary responsibility to all shareholders.
a result of The recent Nasdaq decline is a result of higher interest rates.
a sequence of The Sumerian text was a sequence of incomprehensible symbols.
agree with The Teamsters do not agree with the Republicans on many issues.
among Used when discussing more than two items. He was the finest policeman
among the hundreds of rookies.
as good as/or better than The new software is as good
as or better than anything on the market
as great as The new house looks as great as I had hoped.
attend to (someone) The emergency room doctor
attended to the injured victim.
attribute X to Y/X We attribute the results to the new management.
attributed to Y The extinction of the dinosaurs has been attributed to an asteroid
collision.
B
based on The results are based on a comprehensive ten year study.
begin to He will begin to study twelve hours before the test.
believe X to be Y After seeing the flying saucer, I
believe UFOs to be a real phenomenon.
between Used when discussing two things (if there are more than two, then use
among instead).
He could not decide between Corn Flakes or Raisin Bran.
C
care about How much do business schools care about your score?
centers on + noun The GMAT centers on the knowledge of basic math and
writing/reading skills.
choose to The number of students who choose to go to business school has
increased in the last ten years.
consistent with Your grades are not consistent with your abysmal GMAT scores.
contend that He contends that the GMAT has a cultural bias.
consider + noun How important do you consider the test?
continue + to If you continue to study, you will succeed.
contrast A with B If you contrast A with B, you can see the difference.
convert to You may convert muscle to fat if you study too much.
compare A to B (compare to stresses similarities). The music critic favorably
compared him to Bob Dylan.
compare A with B (compare with stresses differences). Broccoli is good for you
compared with ice cream.
count on + noun He counts on management support. concerned with They are
concerned with investor relations more than actual profitability.
conform to When you work at a new company, you should try to conform to its
corporate culture.
D
decide to We decided to continue.
decide on We decided on the new format.
depend on The global economy depends on improving productivity.
different from The CAT is very different from the paper and pencil GMAT.
difficult to Many students find the CAT difficult to take.
distinguish between X and Y Distinguish between domestic and international
production.
distinguish X from Y Juries must attempt to
distinguish truth from falsehood.
depends on whether Our place in the playoffs depends on whether we win tonight.
E
to be + essential to + noun Speed is essential to success in the Internet
marketplace.
except for He did well on the GMAT, except for the sentence correction questions.
F
flee from The convict fled from the country.
G
grow from Dell Computer grew from a start- up to a Fortune 500 company in less
than fifteen years.
grow out of Needless to say, they quickly grew out of their first office.
H
help + noun + to Their direct business model helped them to grow rapidly.
I
indicate that Dell's recent stock trouble may indicate that their growth will not
continue to be as rapid.
invest in He is too risk-averse to invest in the stock market
identical with His DNA is identical with his twin's.
in contrast to The candidate claims to support tax cuts, in contrast to his prior
statements.
independent from The Federal Reserve Board is supposed to be independent from
political considerations.
indifferent towards Some countries are indifferent towards human rights.
L
leads to Rapid growth often leads to problems.
like Usually only used for direct comparison: He walks
like Joe walks.
localized in Most Internet venture capital is
localized in a few areas of the world.
M
mistake + noun + for I mistook you for an old friend.
modeled after The judicial building is modeled after the Parthenon.
more than ever Companies demand MBA graduates now more than ever.
N
native to There is a unique business culture native to the U.S.
a native of It infects those who are not even a native of America.
need to Living in New York City is an experience everyone needs to try.
to be + necessary + to It is necessary to get a high GMAT score to get into Stanford.
neither...nor Neither Tom nor Sam has the necessary skills to finish the job.
not only...but also Stanford not only has the highest GMAT average, but also the
highest GPA.
P
prohibit from + gerund You are prohibited from using a calculator on test day.
potential to A graduate of a top business school has the potential to make over
$100,000.
R
range from X to Y The GMAT scores at top business schools will range from 650 to
750.
refer to If you have any more questions, you should
refer to a grammar book.
regard as Wharton's finance program is regarded as the finest in the world.
require + noun + to You require a GMAT score to go to most U.S. business schools.
rivalry between X and Y The rivalry between the Boston Red Sox and the New York
Yankees is one of the most celebrated in professional sports.
responsible for The manager is responsible for seven entry level employees.
retroactive to The tax policy change is retroactive to last year.
S
save for Save for William, no one else passed the exam.
save from Many people use business school to save them from dull jobs.
so that So should not be used as an adjective: GMAT preparation is so... boring. Use
it with "that." This guide is designed so that you may raise your score.
subscribe to Business school students should subscribe to the Wall Street Journal.
T
tie to The contract should be tied to concessions.
transmit to The communications system will transmit to anyone within range.
U
used + infinitive Japan used to be the model industrial economy.
to be + used to + gerund After five practice tests, he was used to the GMAT CAT
format.

Word Pairs (Examples)


as…as The movie was as long as it was boring.
more…than
less…than The workshop was more thrilling than anything I'd ever done.
greater….than
not only…but
She was not only exhausted but (also) famished as well.
(also)
The apartment was so expensive that no self-supporting student
so…that
could afford it.
(just) as…so (Just) as it is the duty of employees to contribute to the well-being
of the company, so it is the duty of the company to contribute to
the well-being of its employees.
neither…nor
Neither a borrower nor a lender be.
either…or

Word Usage
Some pairs of words, like fewer and less, are often used incorrectly because
they're treated as synonyms. In fact, there is a solid rule that determines which
one you should use, and the exam will test your ability to decide which is the
correct option. The exam tests four such word pairs with particular frequency, so
memorize the rules pertaining to them if you don't know them already.
1. 1. Fewer/Less
Which of the two following sentences is correct?
1. 1. The company fired no less than fifty employees.

2. 2. The company fired no fewer than fifty employees.


The second sentence is correct. Why? Because you use less when you're talking
about things you can't count (less pollution, less violence) but fewer when you're
talking about things you can count (fewer pollutants, fewer violent acts).
2. 2. Number/Amount
These words follow the same rule as less and fewer. Number is correct when you
can count the thing being described (a number of cars, a number of people) and
amount is correct when you cannot (amount of love, amount of pain).
3. 3. Among/Between
Use between when only two options are available (between the red car and the blue
car) and among when more than two options are available (among the five answer
choices, among the many books).
4. 4. If/Whether
Whether is correct when you're discussing two options (whether to get chocolate or
strawberry ice cream) and if is correct for more than two options (if she should get ice
cream, frozen yogurt, or a cookie).
5. 5. each other/one another
each other – refers to two things (Tom and Linda helped each other up the hill)
one another – refers to 3 or more things (Each member of the family supported one
another)
6. 6. Another/the other
Another – nonspecific other (This is another fine mess you’ve got me into)
Other- specific other (During lunch, I ate one dessert and Linda ate the other one.)

Use of Being
Important! Although many of these absolute phrases could be written with the word being
in them, more formal English (and ETS!) tends not to use being when being is optional. If
you've studied GMAT Sentence Correction for a while, then you know that the word being
raises a big red flag on the test!
Here are some examples:
• • The movie being over, we left the theater.
This sentence could be rewritten like this:
• • The movie over, we left the theater.
People who study for GMAT for a while quickly learn that being is usually wrong.

So I'm guessing you already know that being in an answer choice is wrong more often
than it is right.

This is a good strategy to get you started, but to get over 700 on the GMAT, you really
need to know some of the finer points of GMAT Sentence Correction that relate to the use
of the word

General Grammar definitions


So= therefore
So that=in order to/in order that
That=the fact that

Conjunctions that can join two independent clauses are: and, but, yet, for, or, nor
Don’t start sentences with “Because”

Words requiring “how” (example: know how)


Know
Teach
Learn
Show

Always choose active over passive voice


Active example: Elaine purchased new software for the company
Passive example: New software was purchased for the company by Elaine
Another frequently tested grammatical error is unnecessary use of the passive
voice. It's a good idea to become familiar with this type of error; it appears quite
often in the answer choices of sentence correction questions.
As a reminder, the passive voice is in use when the action of the sentence is
performed on the subject. The active voice is in use when the subject itself
performs the action.
Example:
Active: Allison (subject) went (action) to the store to buy a cake (object).
Passive: The cake (subject) was bought (action) by Allison (object).

Preposition use
“To” or “Of” needs subject + verb NOT object + verb
Examples of subject: she, he, whoever, who, I
Examples of object: her, him, whomever, its, it

Either side of a form “to be” (were, was) must have subjects not objects on either side to
agree.

A verb that ends in –ing is a gerund which is a verb acting like a noun. Try not to use
gerunds if you can help it.

Adverbs, not adjectives modify verbs


Key identifier: Adverbs end in –ly
Correct example: I sure wish I were rich
Incorrect example: I surely wish I were rich

being

There are at least two different situations in which being is often the right answer.
Here is the first example of when being is correct:
When the grammar requires it.
Yes, I'm trying to simplify things here, but the idea is this--many ideas can be expressed
in more than one way. For example, I can say:

I'm afraid of being late.


I'm afraid that I'll be late.
Each has its own emphasis, but the point is that these two structures exist.

Whether we can express ideas in one or more structures is really related to the word
used; in other words, it is idiomatic.

But some idioms allow only one structure. For example:

In addition to being one of the first restaurants to combine Mediterranean and American
tastes, Chez Panisse in Berkeley is also one of the Bay Area's most established
restaurants.

The idiomatic structure in addition to does not have a counterpart that uses a subject and
a verb, so our only option here is to use being, which is grammatically a noun, but is
derived from a verb.

The second example of when being is correct is shown in this example:

There are many reasons to get an MBA, with increased career prospects being the most
important for many MBA applicants.

Technically this part here:

with increased career prospects being the most important for many MBA applicants

is an absolute phrase, but I think it's also helpful just to memorize the pattern:

with + NOUN + being + NOUN COMPLEMENT

Like vs. Such As


Question: What's the difference between like and such as?
Example of the "mistake" that we make in everyday speech: Can you buy me some fruit like
oranges or grapefruit?
How the GMAT Official Guide would explain this mistake:
Using like in this answer choice mistakenly suggests that the utterer of the request does
in fact not want oranges or grapefruit, but rather some other kind of fruit that is similar to
oranges or grapefruit.
In normal English:
like means similar to, and
such as means for example.
Take a look at these examples:
• Can you buy me some fruit like oranges or grapefruit?

In GMATLand, this sentence would mean that you do NOT want oranges or grapefruit; instead,
you'd prefer some fruit similar to oranges and grapefruit. For example, you may want pomelo,
lemons, or limes. Yes, I know this sounds a little crazy, but our goal is to understand what GMAT
is looking for, not what is "correct" English.
• Can you buy me some fruit such as oranges or grapefruit?
Yes, this is what we're supposed to say in GMATLand -- oranges and grapefruit are examples of
the type of fruit we want.
• I would like you to buy such fruit as oranges and grapefruit for me, if you don't mind.
This is simply a variation -- notice how such and as are separated. Separating the two elements
tends to make this pattern a bit harder to see.

What does Question form is obligatory mean?


This simply means that you MUST invert the subject and the verb in this construction. In other
constructions, inversion is optional, but in these constructions, it is required.
For example, you may say:

• • She runs faster than do most of her classmates.


(verb comes before the subject)
or

• • She runs faster than most of her classmates do.


(subject comes before the verb)
BUT

• • Never have I heard such a thing!


CANNOT become

• • XX Never I have heard such a thing! XX

Like vs. Such As


Question: What's the difference between like and such as?
Example of the "mistake" that we make in everyday speech: Can you buy me some fruit like
oranges or grapefruit?
How the GMAT Official Guide would explain this mistake: Using like in this answer choice
mistakenly suggests that the utterer of the request does in fact not want oranges or grapefruit, but
rather some other kind of fruit that is similar to oranges or grapefruit.
In normal English: In GMATLand, like means similar to, and such as means for example. Take a
look at these examples:
• Can you buy me some fruit like oranges or grapefruit?
In GMATLand, this sentence would mean that you do NOT want oranges or grapefruit; instead,
you'd prefer some fruit similar to oranges and grapefruit. For example, you may want pomelo,
lemons, or limes. Yes, I know this sounds a little crazy, but our goal is to understand what GMAT
is looking for, not what is "correct" English.
• Can you buy me some fruit such as oranges or grapefruit?
Yes, this is what we're supposed to say in GMATLand -- oranges and grapefruit are examples of
the type of fruit we want.
• I would like you to buy such fruit as oranges and grapefruit for me, if you don't mind.
This is simply a variation -- notice how such and as are separated. Separating the two elements
tends to make this pattern a bit harder to see.

What's the Difference Between that and which?


After people study GMAT sentence correction for a while, they may ask about the difference
between that and which. I have to say for the record, though, that it is not terribly important to
know the difference between the two to get a high score on the test, but I know that some people
are naturally curious about the difference (people here tend to be overachievers!).
A Tip
But first, a testmagic tip: GMAT almost always (I say almost always because I've seen two
questions that did not follow this rule, but the rule was violated in all five answer choices) wants
you to put a comma before which. In other words, if you see which without a comma before it,
it's probably wrong. After this explanation, you should understand why, but for those of you who
want only the most important information, this is what you need to know.
An Example
Both of these sentences are correct in GMAT land:
• Please go into the room and get me the big book, which is mine.
• Please go into the room and get me the big book that is mine.
Yes, in GMATland, these two sentences have two different meanings.
Both of these sentences would be incorrect in GMAT land:
• X Please go into the room and get me the big book which is mine. X
• X Please go into the room and get me the big book, that is mine. X
Notice the commas--that's what makes all the difference.
The Explanation
Okay, we have in English this weird idea that we need to use different grammar in an adjective
clause (a.k.a. relative clause) depending on whether the information in the adjective clause is
necessary to specifically identify which noun we are referring to.
For example, imagine you have one sister, and you are telling a friend that your sister is coming
to visit you. Since this person is your friend, we can presume that he knows that you have only
one sister.
You utter a sentence like this to your friend in GMATland:
• "My sister, who just graduated from college, is coming to see me."
In GMATland, since your friend (we presume) knows you well and knows that you have only one
sister, this extra bit of information is considered unnecessary to identify which sister it is you are
talking about. It is a sort of "by the way" information--"My sister is coming to see me, and oh, by
the way, she just graduated from college."
Now imagine you have two, three, or even more sisters. Let's imagine that one is a college
professor, another is a webmaster, and this one who is coming to visit you just graduated from
college. If you're talking to your friend, and you say only "my sister," and you do not mention her
name, your friend might not know which sister you are talking about. So you add that extra bit of
information--my sister who just graduated from college--to identify which sister it is you are
referring to.
In this situation, we have just correctly employed a very important grammar rule. Read on.
So, if the person you're talking to, or the person who's reading what you've written, needs that
extra bit of information to know which noun you're referring to, we say that that extra information
is non-restrictive. This word doesn't really describe the function clearly, so many teachers say
that this information is "extra."
On the other hand, if you need that information to know which noun you are talking about, we say
that the information is restrictive. Again, this word is not really a good choice for clarity, and
many teachers use the term "necessary information" instead.
Finally, just to make English a bit more difficult, we have a rule that says we should use a comma
before or after "extra information clauses and phrases," but not with "necessary information
clauses or phrases." The idea here is that the comma represents the slight pause in speech or
change in intonation that a native speaker might use when making such an utterance.
I should point out that both that and which are relative pronouns, i.e., they are grammatically the
same, but their meanings are slightly different.
Now, let's return to our original example sentences:
• Please go into the room and get me the big book, which is mine.
In this sentence, the clause which is mine is "extra" because the information "the big book" is
enough to identify which book it is that you want. We can assume that there is only one big book
in the room.
• Please go into the room and get me the big book that is mine.
In this sentence, the clause that is mine is "necessary" because the information "the big book" is
NOT enough to identify which book it is that you want--it is probably the case that there are
several big books in the room, so I need to add the information "that is mine" to identify which
book it is that I want.
More Examples
A few more examples may help:
• I met with Bill Clinton, who is a lawyer.
The name Bill Clinton is enough to identify which person I'm talking about--who is a lawyer is
therefore extra information.
• I met with the man who is a lawyer.
In this case, "the man" is not enough information to identify which person I'm talking about--who
is a lawyer is therefore necessary information.
• The Sun, which is the only star in our solar system, is the source of heat for Earth.
Again, the name "the Sun" already clearly identifies the noun; therefore, the information in the
adjective clause "which is the only star in our solar system" is extra.
• The star that is at the center of our solar system is called what?
In this case, since we don't have a name here, we don't know which star it is that we are referring
to. Therefore, the information in the adjective clause "that is at the center our solar system" is
necessary.
A good rule to learn is that which requires a comma before it. If you're not sure about why the
comma is necessary, please see this post.

However, GMAT is very tricky at times, and can of course create a question in which this rule
does NOT apply. In fact, I've seen a question that used which without a comma, but ALL the
answer choices violated the rule. I.e., none of the answer choices used that; they all used which
without a comma.

Here's an example of what I mean:

Lucise in San Francisco is a place which anybody can visit.


(A) which anybody can visit
(B) which any person could visit
(C) which no person could not visit
(D) which, if they wanted to, any person could visit
(E) which any person could visit if they so desired

I know some of you will think that this sentence is terrible, but the whole point of GMAT sentence

correction is that we must choose the best answer, NOT the perfect answer.
Critical Reasoning
On the verbal part of the GMAT, you will encounter about 14 Critical Reasoning questions of
various lengths (sometimes you will even need to scroll to read all the answer choices). In 99 per
cent of cases, you will have a short passage with one question right under it. The argument you
meet can be anything from a classical argument to an advertisement or a dialog. The questions
will ask you to manipulate the argument to weaken/strengthen it, find the conclusion, assumption,
explanation, do an inference, supplement a statement, or even tell how its parts are related to
each other. On average, you will have 1:50 for each question, but it is recommended that you try
to stay within 1:30 on CR (Critical Reasoning) questions since you will need to save some time
for Reading Comprehension.

It is recommended that you read through Kaplan's Verbal workbook or the Section of CR in the
Kaplan GMAT book with CD. Both of the books are good for building a solid background; in our
sessions, we will develop further the techniques described in these books, yet will not rely on
them for exercises or anything else.

First of all, Critical Reasoning is ability to reason clearly to evaluate and judge arguments. You
are using this skill a lot during you everyday life while reading newspapers or watching movies.
When you think that the movie is pushing the limit of the Reasonable or the news sounds less
reasonable than the movie that was pushing the limit, you are using your CR skills to produce
these conclusions. Besides the Verbal part on the GMAT, you will also need good argumentative
skills to beat the essays since one of them is to construct an argument, and the other is to
evaluate one. As a rule, GMAT CR questions will ask you to manipulate the argument to
weaken/strengthen it, find the conclusion, assumption, explanation, do an inference or
supplement a statement, etc. Whatever it is that you have to do, you will need 2 things to
succeed: know the basic structure of arguments and clearly understand the argument.

In general, about 80% of GMAT arguments consist of evidence, usually 2 pieces, a conclusion -
the main point of an argument, and an assumption - the bridge between the evidence and
conclusion. The majority of the arguments you encounter on the test will be 3 step arguments:
Evidence1 + Evidence2 = Conclusion.
Example 1. Last week Mike was detained for shoplifting at a groceries store near
his house, but he has been a Christian for 10 years, therefore, the police must have
been wrong accusing him in stealing.
We have here two pieces of evidence: Mike was accused of stealing and that his is a Christian.
The conclusion is that the police are wrong. Therefore, our huge assumption here is that a
Christian could not have stolen anything.

Example 2. There are a lot of mosquitoes outside today, please do not turn on the
light in the room because a lot of them will fly in.
There is no set scheme for structure in GMAT CR, but since the majority of the arguments are
only a few sentences long, the conclusion usually comes in the first or the last sentence.
However, some of the arguments you encounter will not have a conclusion at all or will have just
an implied one.
Let's go through the strategy to approach CR questions; we will cover it today in general and then
will practice some of the crucial steps one by one and then in the third session will consolidate the
approach.

Critical Reasoning Strategy


[We want to warn you immediately that this strategy is not the easiest way to do CR (the easiest
would be read-and-answer), but it lets you get the most questions right spending less time per
correct answer. The bottom line is that it won't be easy to follow this strategy but if you do, it will
reward you]
1. Read the passage (Read it very attentively because in contrast to Reading
Comprehension, there is very little text here and mostly everything is important; try to
read only once. Reread only hard texts). Identify any evidence or premises and the
conclusion. Note, sometimes the conclusion is the question itself. ETS… damn…
they’re tricky.
2. Paraphrase the passage (this a very important step because when you do a paraphrase,
you check whether you understood the passage and at the same time you extract the
skeleton of the argument, making it easier to identify the conclusion and the assumption.
Very often, the paraphrase of the passage will be pretty close to the conclusion. It is not
surprising, since the conclusion is the main point and evidence just supports it.) Your
paraphrase should be as close to the text and as simple as possible so that you would
understand it easily and at the same time could fully trust it. Do not make it too general
nor too detail oriented. When you do a paraphrase, do it in three steps: Evidence1,
Evidence2, and Conclusion; put "therefore" word before you start your conclusion, this
will help you to set it off.
3. Read the question again (now with more understanding of what is being asked; reading
the question 2 times, will also help you to make sure you answer exactly what is stated
and that you understand the question.)
4. Answer before reading the answer choices (Why do this? Two reasons: one, if you can
think of the correct answer or at least the general direction that the answer choice needs
to be, you will identify it among the wrong choices much faster, thus spend less time
reading the answers, which usually take 30 seconds to cover. The second reason is that
often test takers are seduced by the author's wording. One reads a few words that were
used in the passage and the brain identifies this choice with the passage, thus making it
seem more right that it needs to be. The more problems you practice with, the more
chance is you will guess the right answer even before reading it. And there is nothing
more pleasant than seeing YOUR answer choices among the listed. )
5. Go through the answers, first time scan them for YOUR answer choice (usually you will
guess correctly in 60-70% of cases), if you did not find it, reread them more attentively.
6. Draw a grid to eliminate the wrong answers easier. Use "+" for a sure answer, "-" for a
definitely wrong answer choice, and "~" or "?" for an answer that may be right or
questionable. This will help to concentrate only on a few answer choices and will prevent
you from reading same answers several times if you get confused or keep having
troubles locating the right answer.
A - +
B ? -
C - -
D - -
E - +
7. Always remember to think how the answer choice relates EXACTLY to this situation; it
may be out of scope by being too general. E.g. Advertisement:

For sinus pain, three out of four hospitals give their patients Novex.
So when you want the most effective painkiller for sinus pain, Novex
is the one to choose.

Which of the following, if true, most seriously undermines the advertisement's


argument?

(A) --
(B) --
(C) Many drug manufacturers increase sales of their products to hospitals
by selling these products to the hospitals at the lowest price the
manufacturers can afford.
(D) Unlike some competing brands of painkillers, Novex is available
from pharmacies without a doctor's prescription.
(E) --

In this next section, we will be mostly concerned with one of the most important parts of a GMAT
argument, Assumption.

Many CR questions directly ask for an assumption or are based on them, such as weaken and
strengthen questions. Also, assumption of an argument is one of the only parts that we can
influence to destroy or solidify an argument; thus if we can disprove an assumption, we can
negate the whole argument because the conclusion will not make sense. On the other hand, if we
can strengthen our assumption, thus patch a possible hole, we will create a stronger argument
and our conclusion will be more credible. (Sometimes some of the evidence is doubted or is
amended to add new meaning and change the argument, but generally, it is the assumption that
is attacked to destroy an argument. In a case when evidence is completed with more info, we can
still say that it is the assumption that the evidence we had was valid is being destroyed). Thus, to
succeed in CR, it is crucial that you are able to extract an assumption fast; it will save you time on
many questions. Practicing with assumptions does not take too long; 2-4 hours and your skills will
be on top. Also, you will be a better speaker and a more critical writer if you are able to see
assumptions of the writer who argues against you because as we have said, if you can kill an
assumption, you will damage the argument. For example, if we play with Example 1 and add a
piece of evidence that says that Christians sometimes steal, we will not be able to say with
confidence that the police are wrong about Mike being a thief, since evidence proves that
Christians do steal.

We will also cover one of the most important steps of the CR approach, paraphrase.
Paraphrase allows you to check how well you understood the passage as well as to see the
structure stripped. Paraphrase should not take you more than 5-10 seconds after you have read
the passage. Sometimes it is useful to write down some complicated relationship or something
that will help you to understand the passage better, such as a diagram, for example. Other than
that, don't write down anything else, the passage that you read will be so short that you should be
able to remember all the details without writing down anything.
Today we were supposed to learn that the assumption of an argument is
• a bridge (link) between the evidence and the conclusion of an argument
• never stated in the text
• is the most vulnerable element of an argument
• is the only element in the argument that can be influenced
• should be easy for you to find by now

Paraphrase
• Should be brief and take 5-10 seconds
• Will state the main idea of the passage and be close to the conclusion
• Will help you to understand the passage better
• Will reveal the conclusion, evidence, and eventually, the assumption
• Start your conclusion with Therefore

Homework
Work on the section 10 and 11 of the GMAT+ materials available for download at
www.angelfire.com/scifi2/gmatplus/gmat.html, paraphrasing all passages and writing down the
assumptions.
Also, please, underline the conclusions; some of the passages won't have a
conclusion/assumption; in this case, leave out the conclusion and assumption, but still do a
paraphrase.
To check your answers, go to the appendix of the guide:
1) 1) Section 10 can be found in Appendix B
2) 2) Section 11 can be found in Appendix C
Appendix A
Free Scaled down version of information you’ll find in Deltacourse. Deltacourse is 10 times more
thorough and covers more subcategories of Quant question types not covered well in Princeton
or Kaplan.

Arranging Objects

The number of ways of arranging n unlike objects in a line is n! (pronounced ‘n factorial’). n! = n ×


(n – 1) × (n – 2) ×…× 3 × 2 × 1

Example
How many different ways can the letters P, Q, R, S be arranged?
The answer is 4! = 24.

This is because there are four spaces to be filled: _, _, _, _


The first space can be filled by any one of the four letters. The second space can be filled by any
of the remaining 3 letters. The third space can be filled by any of the 2 remaining letters and the
final space must be filled by the one remaining letter. The total number of possible arrangements
is therefore 4 × 3 × 2 × 1 = 4!
The number of ways of arranging n objects, of which p of one type are alike, q of a second type
are alike, r of a third type are alike, etc is:
n!/(p! q! r!) …

Example

In how many ways can the letters in the word: STATISTICS be arranged?
There are 3 S’s, 2 I’s and 3 T’s in this word, therefore, the number of ways of arranging the letters
are:
10! = 50 400
3! 2! 3!

Rings and Roundabouts

The number of ways of arranging n unlike objects in a ring when clockwise and anticlockwise
arrangements are different is (n – 1)!
When clockwise and anti-clockwise arrangements are the same, the number of ways is ½ (n – 1)!

Example

Ten people go to a party. How many different ways can they be seated?
Anti-clockwise and clockwise arrangements are the same. Therefore, the total number of ways is
½ (10-1)! = 181 440
Combinations
The number of ways of selecting r objects from n unlike objects is:
nCr = n!/r! (n – r)!

Example

There are 10 balls in a bag numbered from 1 to 10. Three balls are selected at random. How
many different ways are there of selecting the three balls?
10C3 = 10! = 10 × 9 × 8 = 120
3! (10 – 3)! 3 × 2 × 1

Permutations

A permutation is an ordered arrangement.


The number of ordered arrangements of r objects taken from n unlike objects is:
nPr = n!/(n – r)!

Example

In the Match of the Day’s goal of the month competition, you had to pick the top 3 goals out of 10.
Since the order is important, it is the permutation formula which we use.
10P3 = 10!/7! = 720

There are therefore 720 different ways of picking the top three goals.

Probability
The above facts can be used to help solve problems in probability.

Example

In the National Lottery, 6 numbers are chosen from 49. You win if the 6 balls you pick match the
six balls selected by the machine. What is the probability of winning the National Lottery?
The number of ways of choosing 6 numbers from 49 is 49C6 = 13 983 816 .
Therefore the probability of winning the lottery is 1/13983816 = 0.000 000 071 5 (3sf), which is
about a 1 in 14 million chance

The probability of an event occurring is the chance or likelihood of it occurring. The probability of
an event A, written P(A), can be between zero and one, with P(A) = 1 indicating that the event will
certainly happen and with P(A) = 0 indicating that event A will certainly not happen.

Probability = the number of successful outcomes of an experiment


the number of possible outcomes
So, for example, if a coin were tossed, the probability of obtaining a head = ½, since there are 2
possible outcomes (heads or tails) and 1 of these is the ‘successful’ outcome.

Using Set Notation

Probability can be studied in conjunction with set theory, with Venn Diagrams being particularly
useful in analysis.
The probability of a certain event occurring, for example, can be represented by P(A). The
probability of a different event occurring can be written P(B). Clearly, therefore, for two events A
and B,
P(A) + P(B) - P(AnB) = P(AuB)
P(AnB) represents the probability of A AND B occurring. P(AuB) represents the probability of A
OR B occurring.

Mutual Exclusive Events

Events A and B are mutually exclusive if they have no events in common. In other words, if A
occurs B cannot occur and vice-versa. On a Venn Diagram, this would mean that the circles
representing events A and B would not overlap.
If, for example, we are asked to pick a card from a pack of 52, the probability that the card is red
is ½ . The probability that the card is a club is ¼. However, if the card is red it can't be a club.
These events are therefore mutually exclusive.
If two events are mutually exclusive, P(AnB) = 0, so
P(A) + P(B) = P(AuB)

Independent Events

Two events are independent if the first one does not influence the second. For example, if a bag
contains 2 blue balls and 2 red balls and two balls are selected randomly, the events are:
a) independent if the first ball is replaced after being selected
b) not independent if the first ball is removed without being replaced. In this instance, there are
only three balls remaining in the bag so the probabilities of selecting the various colours have
changed.
Two events are independent if (and only if):
P(AnB) = P(A)P(B)
This is known as the multiplication law.

Conditional Probability

Conditional probability is the probability of an event occurring, given that another event has
occurred. For example, the probability of John doing mathematics at A-Level, given that he is
doing physics may be quite high. P(A|B) means the probability of A occurring, given that B has
occurred. For two events A and B,
P(AnB) = P(A|B)P(B)
and similarly P(AnB) = P(B|A)P(A).
If two events are mutually exclusive, then P(A|B) = 0 .

Example

A six-sided die is thrown. What is the probability that the number thrown is prime, given that it is
odd.
The probability of obtaining an odd number is 3/6 = ½. Of these odd numbers, 2 of them are
prime (3 and 5).
P(prime | odd) = P(prime and odd) = 2/6 = 2/3
P(odd) 3/6
Appendix B

Critical Reasoning: Section 10


Please download section 10 from the gmatclub.com’s CR section
Read through passages, but do not answer the questions, don’t even try. What you need to
concentrate first is paraphrasing and pinpointing assumptions – to check yourself, below
you will find paraphrases and assumptions for each question in chapter 10

Question # Paraphrase Assumption


3 Old study – eating chocolate increases People were not buying chocolate
chances of getting heart disease; new study because they were afraid of heart
– chocolate does not increase heart disease disease
chances; therefore, people will buy more
chocolate

4 As climate of North America got hot and dry This assumption is possible only
large mammals died while small ones after brining in an implied
survived conclusion – change in climate
killed the large animals.
Large mammals were more
sensitive to heat than small ones
5 Higher profits will give higher bonuses, The Industry will not bring profits
therefore, general economic recession year in the year of general economic
will bring lower bonuses than year of profits recession

6 There’s less suitable area for gray wolves, People will keep hunting
however, bringing gray wolves back to the
places where they have been hunted out is
immoral

8 Developed a safer equipment for Swedish Safety improvement should be


market, brought it to US but doesn’t advertize because it will help him
advertise this safety improvement win over it’s competitors

9 Few residents use public buses because Killing subsidy for the public bus
there are enough automobiles, therefore, system will not hurt the suburbs
public bus system should not be subsidized community
14 A survey was conducted, according to is Employees with high ratings are
employees with high ratings ware satisfied the company’s best performing
with the company’s system, therefore, the employees
company’s best performing employees like
the system

16 1970’s – fall in the average annual income N/A


of college graduates as compared to high
school graduates;
1980’s – the average annual income of
college graduates increases while the
number of college graduates did not
decrease

19 Industrialists are accused of intervention in Federal money (expenses and


war to make profit, however, federal profits) and industrialists’ money
expenses for intervention were larger than come from the same source
profits, therefore, the accusation is wrongly
motivated
Appendix C

Critical Reasoning: Section 11


Please download section 11 from the gmatclub.com’s CR section
Read through passages, but do not answer the questions, don’t even try. What you need to
concentrate first is paraphrasing and pinpointing assumptions – to check yourself, below
you will find paraphrases and assumptions for each question in chapter 11

Question # Paraphrase Assumption


1. School board decided to reduce its staff It is possible to determine the level
by laying off the least effective teachers of teachers’ effectiveness
first

2. Applied scientific research is emphasized Basic research does not lead to


because it leads to technological technological advancement
advancement; basic research should be
paid attention to because it’s a base for
applied research

3. Decision to invest in electronic system Competitors are not using the


has cost advantage over nonelectronic electronic system yet and won’t
system, therefore, it will give advantage match it
over competitors

5. One museum sold 30K tickets, in a year The first old museum did not sell
2 more museums opened and together more tickets next year, as compared
the 3 of them sold 80K tickets, therefore, to previous year, and it is due to the
museums were worth the cost two new museums that the number
of tickets sold increased

8. Company needs to cut costs, so offer Reducing costs by firing will not hurt
early retirement first, and then fire others company’s productivity: 50 percent
to have the overall reduction of costs to of managers left will be able to do
50 percent the double work load

9. Need to reduce airport congestion, so Passengers will be willing to switch


send passengers by rapid trains to from air travel to trains
several cities while realizing that it is the
major airport that is congested

12. Received 2000 letters, most of them People who wrote the letters
support him, therefore, most people in represent the opinion of majority
the country support him

13. Government wanted to regulate what can Implied conclusion: tobacco and
be shown in tobacco and alcohol alcohol advertisement should be
advertisement, as a result, these ads banned.
became more inventive and humorous 1. Use of tobacco and alcohol can
be influenced by advertisements
2. The funny and inventive
commercials are more persuasive
than the old ones.

15. Lead contamination dropped: federal N/A – no conclusion


regulation went into effect, but mainly
because there was a drop in the use of
leaded gasoline

17. Decrease in traditional child disease, and Children are more sensitive to rare
at the same time increase in rare infections than adults
infection among children whereas few
adults are affected

18. Plants come from less developed nations The value and benefit from these
without compensation, but coal, oil and plants are the same as from coal,
ores are extracted for payment oil, etc. They are comparable

19. Donates an exhibit which demands a The number of visitors to see the
storage space, a routine conservation, exhibit will not override the financial
therefore, it adds to museum’s expenses costs of keeping the exhibit
and does not help it financially

20. Oil prices are remaining low, therefore, Oil and natural gas prices are
natural gas prices will also be low interconnected
Appendix D
General Strategies for Reading Comprehension, by Stephen Bolton
1. Try to read the whole text of the passage once, if possible. Many people think you
should just skim the passage or read the first lines of every paragraph, and not to read
the passage. We believe this is an error: if you misunderstand the main idea of the
passage, you will certainly get at least some of the questions wrong. Give the passage
one good read, taking no more than 3 minutes to read all of the text. Do not read the
passage more than once – that wastes too much time. If you have not understood it
completely, try to answer the questions anyway. Note: this point of reading the whole
passage is important for test-takers whose first language is not English, provided that
they can read the passage in 3 minutes or less.

2. Make brief notes on the text on your scrap paper. As we will see below in greater
detail, you should write down a couple of words on A) the Main Idea or Primary Purpose,
B) Organization/Structure of the passage, and C) the Tone or Attitude of the author (if
applicable). You just need a few words for each of these areas, and altogether it should
not take longer than 30 seconds to write down.

3. Remember that the tone or attitude of the passage is usually respectful and
moderate, never going to extremes of praise nor criticism. ETS obtains its Reading
Comprehension passages from real articles about real academics and professionals. So
the tone of the articles, even when there is criticism in the passage toward an academic
or her work, is always balanced and moderate. In the same vein, articles that deal with
minorities or ethnic groups are almost always positive and sympathetic.

4. Look out for structural words that tell you the important ideas or transitions in a
passage.

Continue the Idea Words


Similarly
Moreover
Additionally
In the same way
Likewise

Conclusion Words
Thus
Therefore
Hence
So
In summary
In conclusion

Contradiction or Contrast Words


Neverthless
Nonetheless
However
But
Although
Though
Even though
Notwithstanding
Yet
Despite
In spite of
On the one hand…on the other hand
While
Unlike

5. Go back to the text of the passage for the answers. Many test-takers fail to return to
the text of the passage to look for the correct answers. They rely solely on their memories
and understanding of the passage after having read or skimmed it. Wrong. ETS is
counting on that. Go back to the text to look for information to answer the questions. Nine
times out of ten, the answer lies within the passage.

Of the 6 most important types of questions for Reading Comprehension, we will first look at Main
Idea/Primary Purpose Questions, and the strategies we can use to answer them.
Main Idea/Primary Purpose Questions
Many people believe there is no difference between the main or central idea of the passage and
the primary purpose of the author of the passage. This is simply not true. Let's take a look at the
subtle but important difference between them:
Main Idea
The question might look something like this:

"Which of the following best states the central idea of the passage?"
"Which of the following most accurately states the main idea of the passage?"
"Which of the following is the principal topic of the passage?"
"The main topic of the passage is...."

Primary Purpose
The question might look like this:

"The primary purpose of this passage is to..."


"The primary purpose of the passage as a whole is to..."
"The primary focus of this passage is on which of the following?"
"The main concern of the passage is to..."
"In the passage, the author is primarily interested in...."
"The passage is chiefly concerned with..."

Strategy:
Main Idea: Look in the first and last paragraphs for the main idea. Any conclusion words like
therefore, thus, so, hence, etc. that you see are most likely introducing the main idea. The correct
answer will say the same thing as it says in the text, but using different words. The Main Idea is
not always stated explicitly in the passage – in fact, more likely than not, it is not stated explicitly.
Therefore, in order to answer this type of question when it is more implicit:
1. Re-read the first line of every passage, and the last line of the first and last paragraphs.
This should give you the general structure or outline of the argument, with which you can
answer the Main Idea question.
2. After determining the general structure or content of the argument, eliminate answer
choices that are too broad or too specific, i.e. answer choices that go beyond the content
of the passage, or that deal with content only discussed in one paragraph of the passage.
3. Make brief notes – a couple of words- regarding the Main Idea on the text on your scrap
paper while you read.
Primary Purpose: What is the author trying to do? What is his intention? If he is evaluating a
theory, then the answer could be something like "Discuss an interpretation". Note that the correct
answer would deal with "an interpretation", because the author is only dealing with one theory. If
the Primary Purpose is to criticize 2 new books, then his intention or his primary purpose might be
to "Critique new studies". Again, as in Main Idea questions, re-read the first line of every passage,
and the last line of the first and last paragraphs. This should give you the general structure or
outline of the argument, with which you can answer the Primary Purpose question.
Note: A good main idea or primary purpose does not go beyond the scope of the passage, nor
does it limit itself to discussing only one part of the passage.

What is the primary purpose of this passage?


discuss the importance of the television program Star Trek for the international space
A)
program
B) discuss important theoretical work concerned with faster-than-light space travel.
C) explore a dispute among theoretical physicists regarding the uses of space flight
D) describe the possible uses of space-warping material
E) explain how a space-warping bubble would work in the real world

Explanation
This is a Primary Purpose question, so we have to determine what the author is trying to do or
say in this passage. So, let's read the first and last lines of the passage in order to get an idea of
the primary purpose. The first line says "Great news for Star Trek fans: warp drives that can
propel starships around the Galaxy faster than the speed of light may be possible after all--with a
little help from Dr Who." The last line is a quote by a physicist that says "Of course, there are still
some basic questions--like how does one go about constructing this Tardis space-time--but it puts
the concept of space warps back on the agenda." From both these sentences, we get the idea of
space travel, faster than light travel and space warps – maybe this is a discussion of faster than
light space travel. Does that match what you have already read? Yes, basically this is a
discussion of the theoretical state of play in the area of faster-than-light space travel. Do any of
the 5 answer choices match that? Yes – B, even if the wording is somewhat different from how we
are wording it, the idea is almost exactly the same. B is the answer.
Another way of getting to the answer is through elimination of obviously incorrect answer
choices. We can eliminate A because the author mentions the popular science fiction program
Star Trek merely to introduce the idea of faster-than-light travel, and nothing more. C is a stronger
possibility because the second paragraph of the passage does discuss some disagreement
among physicists about the possibility of creating a warp-drive, but in the same paragraph the
theoretical dilemma seems resolved. Moreover, since the author only discusses this in one
paragraph, it cannot be the primary purpose of the entire passage. We can eliminate D because
the author does not go into detail discussing the uses of space-warping material. And we can
discard E because the author does not really go into how the space-warping bubble would work
in the real world.
Title Questions
by Stephen Bolton, 20th August, 1999
Title questions are very similar to Main Idea questions, though are less common. Though some of
the example paassage we use in this tutorial and in the Practice Section are from the New
Scientist, and therefore have titles, the passages in the real GMAT will not have titles. The
question might look like this:

"Which of the following titles best summarizes the passage as a whole?"

Strategy:
Treat this as a Main Idea question. A good title sums up the central idea of a passage. Therefore,
in order to answer this type of question:

1. Look in the first and last paragraphs for the main idea. Any conclusion words like
therefore, thus, so, hence, etc. that you see are most likely introducing the Main
Idea/Title. The correct answer will say the same thing as it says in the text, but using
different words.
2. Re-read the first line of every passage, and the last line of the first and last paragraphs.
This should give you the general structure or outline of the argument, with which you can
answer the Title question.
3. Make brief notes – a couple of words- regarding the Title on the text on your scrap
paper while you read.
4. After determining the general structure or content of the argument, eliminate answer
choices that are too broad or too specific, i.e. answer choices that go beyond the content
of the passage, or that deal with content only discussed in one paragraph of the passage.

What would be an appropriate title for this passage?


A) Constructing The Tardis
B) How To Make Space-Warping Material
C) Bubbles In Space-Time
D) Faster-Than-Light Travel: A Possibility?
E) Debate On The Uses of Space Travel

Explanation
This passage actually already has a title, "Warp Factor One". But we have to look for another title
possibility, one that would be most like the Main Idea of the passage. We look at the first and last
paragraphs, and since the Main Idea is that researchers now feel that faster-than-light travel
maybe more than mere fantasy, we can find the correct answer choice. Does any answer choice
corrspond to this idea? Yes- answer D, which is the correct answer.
We can also find the correct answer through elimination. There is nowhere in the passage where
it discusses building Dr. Who's Tardis (pity!), so we can eliminate A. Nor does it tell us how to
make space-warping material. Eliminate B. While bubbles in space-time are discussed at some
length in one of the paragraphs, we cannot say this is the main concern of the passage, and thus
should eliminate C. And nowhere are the uses of space travel discussed, so discard E.

Specific Detail or Target questions are probably the most common types of questions, and the
easiest to answer. The question might look like this:

"According to the passage,...."


"The passage states that ...."

Strategy
The Specific Detail or Target that we are looking for could be a Line Number, or a Name or Date.
Go to the Line Number or Name or Date, and then read several lines above and below it. Find the
answer choice that basically says the same thing as in the passage, though usually with different
words or word order.

According to the passage, Pfenning and Ford


A) demonstrated conclusively the impossibility of faster-than-light travel
B) explored the possibility of bubbles that warp space
C) supported the work of Alcubierre
D) work at of the Institute for Theoretical Physics at the Catholic University of Leuven
E) suggested that a warp drive was not physically possible

Explanation
This is a Specific Detail/Target question, and therefore we look for the Name, Line Number, or
Date that will help us. In this case, the detail consists of the names Pfenning and Ford. We scan
the text, starting from the top of the passage, looking for the names Pfenning and Ford. We find
them in only place, at the beginning of the second paragraph. We read a couple of lines above
the names, and keep reading until a few lines after the names. It says "But in 1997 Michael
Pfenning and Larry Ford at Tufts University in Medford, Massachusetts, apparently killed this
ingenious idea by showing that it needed far more than the entire energy content of the Universe
to work (This Week, 26 July 1997, p 6)". The line after that says the research of another physicist
then resurrected the possibility of FTL travel, negating the implications of the research of
Pfenning and Ford. Now we can answer the question. Do any of the answer choices match the
information given around the target area? Yes- E.
Let's also eliminate. If we re-read what the passage says about Pfenning and Ford, we can
eliminate B, C, and D. None of them are supported by the information in the passage, so let's
eliminate all of them without wasting too much time and with a minimum of fuss. A is tougher to
eliminate. From the sentence that mention Pfenning and Ford, it seems their work does rule out
the possibility of a space-warp drive. But if we read the next line, it says another researcher said it
was indeed possible. So the Pfenning and Ford could not have "conclusively" demonstrated the
impossibity of the FTL drive.

This is probably the most difficult type of Reading Comprehension problem. The question might
look like this:
"It can be inferred that the author makes which of the following assumptions?"
"Which is an assumption underlying the last sentence of the passage?"
"Which of the following, if true, would most strengthen the hypothesis mentioned in lines
17-19?"
"With which of the following statements regarding chaos theory would the author be most
likely to agree?"

Strategy:
1. First, treat this type of problem as a Specific Target question. Look for a target in the
question, find it in the text, and then look above and below it. Often you do not have to
infer very much, the answer remains within the text.
2. If the answer must be inferred and is not stated explicitly within the text, then choose
the answer choice that can be inferred or assumed from the information given. Again, you
should not have to infer very much – only one or two logical steps removed from the
information in the passage.
3. Make sure that the answer choice you decide on does not violate or contradict the Main
Idea of the passage - if it does, the answer choice is probably wrong.

It can be inferred that a house with the propeties of the bubble mentioned in the passage
A) would be larger on the inside than on the outside
B) could move faster than the speed of light
C) might be very energy efficient
D) could move through time
E) would eventually fold in on itself and be destroyed
Explanation
First, let's try to deal with this question as a Specific Target problem. Is there a target in the
question? Yes – the bubble. The bubble is first mentioned at the end of the second paragraph,
and then discussed at length throughout the third paragraph. Remember, we have to look above
and below that target area (as well as read the target area again), so quickly go through the
second, third, and first part of the fourth paragraph.
When you are finished, look at the answer choices. Can any of them be inferred from the
information given in the target area? Well, we could eliminate C, D, and E for simply not being
supported by the information given in the passage. B – maybe, but a house moving through time
seems pretty silly. But in the fourth paragraph the author talks about the Tardis, "which looked like
a police box but had a spacious interior". Big on the inside, small on the outside. Is that like our
house? Yes- answer A. As well, we can choose A because it does not go against or contradict the
Main Idea in this case, which if it had, would have made it necessary to eliminate. So choose A.

The question might look like this:


"The author's attitude towards Morgan's theory could best be described as one of ..."
Strategy:
Look for descriptive words, adjectives or adverbs, that could tell you the author's attitude. For
example, the words unfortunately or flaw suggest a negative connotation, while strength or
valuable emphasize the positive. Make brief notes – a couple of words- regarding the Tone of the
text on your scrap paper while you read. Additionally, keep in mind that the author's attitude
toward a theory, book, or ethnic group will almost always be respectful, even when somewhat
critical.

The author's attitude towards Miguel Alcibierre's theory could best be described as one of
A) Admiration
B) mild skepticism
C) unbridled scorn
D) Dismay
E) complete objectivity

Explanation
Since this is a Tone/Attitude question, we must look in the passage for descriptive words that tell
us what the author thinks of Alcibierre and his theory. In the second paragraph the author call's
Alcibierre's theory "this ingenious idea". This is positive, and the only positive answer choice is A.
A is the correct answer. As well, if we could not find the tone so easily, we could also eliminate C
and D at the very least, for being too extreme.
The question might look like this:
"Which of the following best describes the organization of the passage?"
"Which of the following best describes the organization of the first paragraph of the
passage?"
"One function of the third paragraph is to...."
Strategy:
Re-read the first line of every passage, and the last line of the first and last paragraphs. This
should give you the general structure or outline of the argument, with which you can answer the
question. Remember to make brief notes about the structure of the text on your scrap paper. If
you are looking for the organization of one paragraph, read the first and second sentence of the
paragraph. That will give you a rough idea of what is the structure or organization of the
paragraph.
Which of the following best describes the organization of the second paragraph of the passage?
A) Two investigations that support Alcubierre's theory are introduced
B) Possible objections to the uses of the warp drive are present, and then refuted
An objection to the practicality of the theory is raised, and then another work is cited to
C)
shore up the applicability of the original theory
A work of theoretical physics that supports Alcubierre's theory is raised, and then another
D)
that refutes it is presented
E) Alcubierre's theory is analyzed by a panel of several eminent physicists

Explanation
Read the first sentence of the paragraph: "But in 1997 Michael Pfenning and Larry Ford at Tufts
University in Medford, Massachusetts, apparently killed this ingenious idea by showing that it
needed far more than the entire energy content of the Universe to work (This Week, 26 July
1997, p 6)". Then read the second sentence: "Now Chris Van Den Broeck of the Institute for
Theoretical Physics at the Catholic University of Leuven, Belgium, has resurrected Alcubierre's
proposal". So if we out those two sentences together, and in different words, first the usefulness
of Alcubierre's theory is questioned by two researchers, then the theory is validated by yet
another researcher. Which of the answer choices is closest to this? C. None of the other answer
choices follow the organizational pattern of the paragraph – they reverse it, or are completely
dissimilar. C is the only possible answer.

1. Read the whole text of the passage once.


2. Make brief notes about the text on your scrap paper.
3. Remember that the tone or attitude of the passage is usually respectful and moderate,
never going to extremes of praise nor criticism.
4. Look out for structural words that tell you the important ideas or transitions in a
passage.
5. Go back to the text of the passage for the answers to specific questions.

Transaction Confirmation # VQCA0A741267


(Recording this number is optional)

A receipt email has been sent to bijitendu@yahoo.com

Click Below
Complete GMAT Prep Course (29.95)
Your payment has been routed and processed in a secure environment.

Login Instructions:

Step 1:
To return to the prep course in the future, log in through the 800score.com home page. Click the "Login"
link on the top menu bar.
Step 2:
Enter the name and email address you used to place your order, then hit the continue button. If necessary,
you may also use your confirmation number to login.

Login Information:
Bijitendu Sarma
User ID :
Sarker
bijitendu@yahoo.co
Email address :
m
Confirmation Number : VQCA0A741267

You might also like